4
$\begingroup$

Let $A\subset\mathbb{R}^p$ and $B\subset\mathbb{R}^q$, it’s not difficult to show that $$m^*(A\times B)\leq m^*(A)\cdot m^*(B)$$, where $m^*()$stands for the outter measure in Lebesgue meaning.

If A and B are measurable, then "=" holds. My question is whether "=" holds for all of/ none of/ some of the non-measurable A and B?

Construction of A and B in these different cases is needed.

$\endgroup$

1 Answer 1

4
$\begingroup$

For simplicity, assume $p = q = 1$ and $A, B \subseteq [0, 1]$. Let $\mu_k, \mu_k^{\star}$ denote the $k$-dimensional Lebesgue measure, outer measure respectively. Let $\{U(n) : n \geq 1\}$ be a sequence of decreasing open sets each containing $A \times B$ such that $\mu_2(\bigcap_n U(n)) = \mu_2^{\star}(A \times B)$. For every $x \in A$, $B \subseteq U_x(n) = \{y: (x, y) \in U(n)\}$ so $\mu_1(U_x(n)) \geq \mu_1^{\star}(B)$. So the (measurable) set $\{x : \mu_1(U_x(n)) \geq \mu_1^{\star}(B)\}$ contains $A$ and hence has measure $\geq \mu_1^{\star}(A)$. By Fubini's theorem, $\mu_2(U(n)) \geq \mu_1^{\star}(A)\mu_1^{\star}(B)$. It follows that $\mu_2^{\star}(A \times B) \geq \mu_1^{\star}(A)\mu_1^{\star}(B)$.

$\endgroup$

Your Answer

By clicking “Post Your Answer”, you agree to our terms of service and acknowledge you have read our privacy policy.

Not the answer you're looking for? Browse other questions tagged or ask your own question.